I need help pleaseee

I Need Help Pleaseee

Answers

Answer 1

The matrix formed by performing the row operation 4R₂ + R₃ — R₃ will result to R₁ = [-4 0 1 5], R₂ = [-6 5 -1 8], and R₃= [-33 24 -9 32]

What is a matrix row operation

Row operations are a set of operations that can be performed on the rows of a matrix in order to transform it into a row equivalent matrix, which has the same solution set as the original matrix.

performing the row operation 4R₂ + R₃ — R₃, we have;

4(-6) + (-9) = -33 {row 3 column 1}

4(5) + 4 = 24 {row 3 column 2}

4(-1) + (-5) = -9 {row 3 column 3}

4(8) + 0 = 32 {row 3 column 4}

Therefore, the matrix formed by performing the row operation 4R₂ + R₃ — R₃ will give a new one with R₁ = [-4 0 1 5], R₂ = [-6 5 -1 8], and R₃= [-33 24 -9 32]

Read more about matrix here:https://brainly.com/question/28979057

#SPJ1


Related Questions

determine the values of the six trigonometric functions of (- pi / 2 )
with work shown please.

Answers

Answer:

[tex]\circ \quad sin\left(-\dfrac{\pi}{2}\right) = -1\\\\[/tex]

[tex]\circ \quad \cos \left(-\dfrac{\pi }{2}\right)= 0\\\\[/tex]

[tex]\circ \quad \tan \left(-\dfrac{\pi }{2}\right) = -\dfrac{1}{0}[/tex]   is not defined

[tex]\circ \quad \csc\left(-\dfrac{\pi }{2}\right) =-1\\\\[/tex]

[tex]\circ \quad \sec\left(-\dfrac{\pi }{2}\right) = \dfrac {1}{0}\\[/tex]  is not defined

[tex]\circ \quad \cot\left(-\dfrac{\pi}{2}\right) =\:0[/tex]

Step-by-step explanation:

[tex]\text{Values of trig functions for $-\dfrac{\pi}{2}$ are: }\\\\[/tex]

[tex]\circ \quad sin\left(-\dfrac{\pi}{2}\right) = - sin\left(\dfrac{\pi}{2}\right) = -1\\\\[/tex]

[tex]\circ \quad \cos \left(-\dfrac{\pi }{2}\right)=\cos \left(\dfrac{\pi }{2}\right) = 0\\\\[/tex]

[tex]\circ \quad \tan \left(-\dfrac{\pi }{2}\right) = \dfrac{sin\left(-\dfrac{\pi}{2}\right)}{cos\left(-\dfrac{\pi}{2}\right) } = \dfrac{-1}{0}[/tex]
This is undefined since division by zero is undefined

[tex]\circ \quad \csc\left(-\dfrac{\pi }{2}\right) = \dfrac{1}{sin\left(-\dfrac{\pi }{2}\right)} = \dfrac{1}{-1} =-1\\\\[/tex]

[tex]\circ \quad \sec\left(-\dfrac{\pi }{2}\right) = \dfrac{1}{\cos\left(-\dfrac{\pi }{2}\right)} = \dfrac {1}{0}[/tex]
This is undefined

[tex]\circ \quad \cot\left(-\dfrac{\pi}{2}\right) = \dfrac{cos\left(-\dfrac{\pi }{2}\right)}{sin\left(-\dfrac{\pi }{2}\right)}\:=\:\dfrac{0}{-1\:}=\:0[/tex]

The strength of a beam is proportional to the width and the square of the depth. A beam is cut from a cylindrical log of diameter d = 24 cm. The figures show different ways this can be done. Express the strength of the beam as a function of the angle in the figures. (Use k as your proportionality constant.)

Answers

The strength of the beam is proportional to the width and the square of the depth, so we can express it as Strength = k * width * depth²

where k is the proportionality constant.

In the given figures, the beam is cut from a cylindrical log of diameter d = 24 cm, which means the maximum width and depth of the beam are limited by this diameter. We can see from the figures that the width and depth of the beam change depending on the angle at which it is cut from the log.

In Figure 1, the width and depth of the beam are equal, so we can express the strength of the beam as:

Strength = k * width * depth² = k * x² * (d/2 - x)²

where x is the width and depth of the beam.

In Figure 2, the width and depth of the beam are not equal, so we need to express them separately. Let w be the width and d be the depth, then we can express the strength of the beam as:

Strength = k * width * depth² = k * w * d² = k * ((d/2)sinθ)² * ((d/2)cosθ)²

where θ is the angle at which the beam is cut from the log.

In Figure 3, the width and depth of the beam also vary with the angle θ. We can express the width and depth as follows:

w = d sinθ

d = (d/2)cosθ

Then we can express the strength of the beam as:

Strength = k * width * depth² = k * (d sinθ) * [(d/2)cosθ]²

Therefore, depending on the angle at which the beam is cut from the log, the strength of the beam can be expressed using different equations, but in all cases, the strength is proportional to the width and the square of the depth, with a proportionality constant k.

To learn more about strength of a beam click on,

https://brainly.com/question/1553947

#SPJ1

Complete question is:

The strength of a beam is proportional to the width and the square of the depth. A beam is cut from a cylindrical log of diameter d = 24 cm. The figures show different ways this can be done. Express the strength of the beam as a function of the angle in the figures. (Use k as your proportionality constant.)

Jermaine invested $3,200 in a defined-contribution account. Assuming he is in a 20% marginal tax bracket, how much did he lower his income taxes with the investment

Answers

Answer:

  $640

Step-by-step explanation:

You want to know the income tax reduction resulting from investing $3200 in a defined-contribution account when the marginal tax rate is 20%.

Tax exempt

The money invested in a defined-contribution account is not subject to income taxes when it is invested. The tax savings is ...

  20% × $3200 = $640

Jermaine lowered his taxes by $640.

Which expression is equivalent to (x-3)(2x^(2)-3x-1)

Answers

The "Expression" which is considered equivalent to this expression "(3x-1)-2(x+2)' is (c) x-5.

In mathematics, an algebraic expression is a combination of numbers, variables, which are joined by arithmetic operations (such as addition, subtraction, multiplication, and division).

We have to find the equivalent-expression for (3x-1)-2(x+2);

⇒ (3x-1)-2(x+2),

⇒ (3x - 1) - 2x - 4,

Combing the like-terms together in the above expression,

We get,

⇒ 3x - 2x -1 - 4,

⇒ x - 5,

Therefore, the correct equivalent expression is Option (c).

Learn more about Expression here

https://brainly.com/question/29143314

#SPJ1

The given question is incomplete, the complete question is

Which of the following expression is equivalent to (3x-1)-2(x+2)?

(a) x+3

(b) x+1

(c) x-5

(d) x-3

Help Please The sum of two even numbers is even. The sum of 6 and another number is even. What conjecture can you make about the other number?
A) The other number is odd.
B) The number is even.
C) Not enough information.
D) The number is 8.

Answers

Answer:

the number is even

Step-by-step explanation:

an even number plus an even number will aalways be even. adding 6 to an odd number will never be even. Therefore the number will be even

Using the GCF, what is the factored form of 75 - 3x?

please i need help!!!
u will get 100 points!!!​

Answers

The GCF (Greatest Common Factor) is the largest number that divides evenly into two or more numbers. In this case, we can find the GCF of 75 and 3 by listing their factors and finding the largest one they have in common. The factors of 75 are 1, 3, 5, 15, 25, and 75. The factors of 3 are 1 and 3. The largest factor they have in common is 3. Therefore, the GCF of 75 and 3 is 3.

We can use the GCF to factor the expression 75 - 3x by dividing each term by the GCF and writing the expression as a product. In this case, we can divide both terms by 3 to get 75/3 - (3x)/3, which simplifies to 25 - x. We can then write the original expression as a product by multiplying this simplified expression by the GCF: 3(25 - x).

Therefore, the factored form of 75 - 3x using the GCF is 3(25 - x).

Answer:

3(25-x)

Step-by-step explanation:

First we find the GCF of 75 and -3x, which is 3

3 is the highest number we can divide both 75 and -3

Hence, the answer is 3(25-x)

A population of values has a normal distribution with a mean of 246 and a standard deviation of 89.7. You intend to draw a random sample of size m = 158.

Answer the following, rounding your answers to three decimals where appropriate.

Find the probability that a sample of size n = 158 is randomly selected with a mean greater than 242.4.
P(M>242.4) = ???

Answers

The probability that a sample of size n = 158 is randomly selected with a mean greater than 242.4 is 0.751.

How to calculate the probability

z = (x - μ) / (σ/√n)

where x is the sample mean.

Substituting the values, we get:

z = (242.4 - 246) / (89.7/√158) ≈ -0.670

We want to find the probability of obtaining a sample mean greater than 242.4, which is equivalent to finding the probability of obtaining a standardized sample mean greater than z = -0.670. We can use a standard normal distribution table or calculator to find this probability.

P(z > -0.670) ≈ 0.751

Learn more about probability on

https://brainly.com/question/24756209

#SPJ1

Line g has an equation of y = 2x + 1. Line h includes the point (-5, 2) and is perpendicular
to line g. What is the equation of line h?
Write the equation in slope-intercept form. Write the numbers in the equation as simplified
proper fractions, improper fractions, or integers.
Submit

Answers

The equation of line h is: y = (-1/2)x - 1/2

Slope-intercept form is y=mx+b, where m is the slope and b is the y-intercept. Line g’s equation is already in the correct form. If line g has a slope m=ab, then line h perpendicular to it will have a slope m⊥=−ba.

2/1 = 2, so the reciprocal is 1/2. It will also be negative to become: y= (-1/2)x + b.
We want to find the y-intercept, b.

Plug in the coordinate that line h passes through (-5,2) for the (x,y) values in the equation.
2 = (-1/2)(-5) + b
2 = 5/2 + b
Subtract 5/2 from both sides:
Note that 2 is equal to 4/2, which makes it easier to deal with.
(4/2) - (5/2) = b

Simplify:
-1/2 = b.

Put all values into line h’s equation, and y = (-1/2)x - 1/2.

f(x)=4x-3
g(x)-3x+3
find f(9)/g(9)

Answers

Hello, hope this helps, goodluck!

Cindy was asked by her teacher to subtract 3 from a certain number and then divide the result by 9. Instead, she subtracted 9 and then divided the result by 3, giving an answer of 43. What would her answer have been if she had worked the problem correctly?

Answers

Using mathematical operations, Cindy's answer would have been 15 if she had worked the problem correctly by following her teacher's instructions.

How is the correct value determined?

The correct value can be determined by reversing Cindy's calculations to find the certain number as follows:

(x - 9) ÷ 3 = 43

x = 43 x 3 + 9

x = 138

With the certain number, x = 138, the correct mathematical operations can be performed as follows:

Correction Operation:

(138 - 3) ÷ 9

135 ÷ 9

= 15

Thus, Cindy's correct answer would have been 15 if she had performed the correct mathematical operations, according to her teacher's instructions.

Learn more about mathematical operations at https://brainly.com/question/4721701.

#SPJ1

Identify the null and alternative hypotheses
Statistics!

Answers

The null and the alternate hypothesis are:

H₀: p ≤ 0.5H₁: p > 0.5

How to write the hypothesis

In hypothesis testing, we take the null hypothesis (H₀) and the alternative hypothesis (H₁). The null hypothesis is the opposite of the claim , while the alternative hypothesis states  the claim.

In this case, the null hypothesis is that a majority of adults would not erase their personal information online if they could (i.e., 50% or less of them would do so). The alternative hypothesis is that a majority of adults would erase their personal information online if they could (i.e., more than 50% of them would do so). In symbolic form:

H₀: p ≤ 0.5

H₁: p > 0.5

Read more on hypothesis here:https://brainly.com/question/606806

#SPJ1

Suppose a polynomial function of degree 4 with rational coefficients has the given numbers as zeros. Find the other zeros.
-3, √3, 13/3
The other zeros are
(Use a comma to separate answers.)

Answers

Answer:

{-3, √3, -√3, 13/3}

Step-by-step explanation:

Since the polynomial has rational coefficients, any irrational zeros must come in conjugate pairs. So, if √3 is a zero, then so is its conjugate, -√3.

We can write the polynomial with these zeros as:

p(x) = a(x + 3)(x - √3)(x + √3)(x - 13/3)

where a is some constant coefficient. Multiplying out the factors, we get:

p(x) = a(x + 3)(x^2 - 3)(x - 13/3)

To find the remaining zeros, we need to solve for x in the expression p(x) = 0. So we set up the equation:

a(x + 3)(x^2 - 3)(x - 13/3) = 0

This equation is true when any of the factors is equal to zero. We already know three of the zeros, so we need to solve for the fourth:

(x + 3)(x^2 - 3)(x - 13/3) = 0

Expanding the quadratic factor, we get:

(x + 3)(x - √3)(x + √3)(x - 13/3) = 0

Canceling out the (x - √3) and (x + √3) factors, we get:

(x + 3)(x - 13/3) = 0

Solving for x, we get:

x = -3 or x = 13/3

Therefore, the other zeros are -3 and 13/3.

The complete set of zeros is {-3, √3, -√3, 13/3}.

Hope it helps^^

15 POINTS!! ASAP TYYY

Answers

Answer:

A

Step-by-step explanation:

She earns $15 per poinsettia.

James answered 14 lems correctly
an a 16-item quiz. what percentage did he answer correctly

Answers

Answer:

14/16 = 0.875

A meter in a taxi calculates the fare using the function f(x) = 2.56x + 2.40. If x represents the length of the trip, in miles, how many miles can a passenger travel for $20?

Answers

A passenger can travel 6.875 miles for $20 using the given taxi fare function.

To determine how many miles a passenger can travel for $20 using the taxi fare function f(x) = 2.56x + 2.40, we need to set up an equation and solve for x.

The equation we need to solve is:

2.56x + 2.40 = 20

To solve for x, we can start by subtracting 2.40 from both sides of the equation:

2.56x = 17.60

Next, we can divide both sides by 2.56:

x = 6.875

This calculation assumes that the fare is based solely on distance and that there are no additional fees or charges.

To learn more about function click on,

https://brainly.com/question/31498912

#SPJ1

For the following exercises,
(a) find the angle between 0 and 2π in radians that is coterminal with the given angle.
(b) find the reference angle of each one.

a)32π/13 b)-17π/6

Answers

The reference angle of given angles are -6π/13 and -5π / 6

To get coterminal angles, you simply have to add or subtract 2π.

a) 32π/13 =

= -32π/13 + 2π

= -32π + 26π / 13

= -6π/13

b) -17π/6 + 2π =

= -17π + 12π / 6

= -5π / 6

Hence the reference angle of given angles are -6π/13 and -5π / 6

Learn more about coterminal angles, click;

https://brainly.com/question/29133154

#SPJ1

Ignatio and his two friends each bought a ticket to play lazer tag and each spent $15 on game tokens at Fun Mountain. Let t represent the cost of a lazer tag ticket. Enter an expression that represents the total amount that Ignatio and his friends spent.

Answers

The expression that represents the total amount that Ignatio and his friends spent is 15t


Calculating the expression that represents the total amount

From the question, we have the following parameters that can be used in our computation:

Each spent $15 on game tokens at Fun MountainEach cost = t

After spending on t games, we have

f(t) = Rate * t

Substitute the known values in the above equation, so, we have the following representation

f(t) = 15 * t

Evaluate

f(t) = 15t

Hence, the expression that represents the total amount that Ignatio and his friends spent is 15t

Read more abiut linear relation at

https://brainly.com/question/30318449

#SPJ1

Girish left his home at 7 45 and travelled to Muar, which was 120 km away. He arrived
at Muar 2 h later. After staying 1 2/3 hours in Muar, he travelled from Muar back to his home
along the same route at a speed which was 20 Km/h slower than his previous speed.
At what time did Girish reach home?

Answers

Answer:

2:35

Step-by-step explanation:

it takes 2 hours to get to Muar so 9:45

he spends 1 hour 40 in muar so 11:35

he went at 60 Km/h before so he returns at 40Km/h so it takes 3 hours so 2:35

The teacher gave a true and false quiz where P(true) = 0.5 for each question. Interpret the likelihood that the first question will be true.

Likely.
Unlikely.
Equally likely and unlikely.
This value is not possible to represent probability of a chance event.

Answers

The likelihood that the first question will be true is equally likely and unlikely.

Interpreting the likelihood that the first question will be true.

From the question, we have the following parameters that can be used in our computation:

The teacher gave a true and false quiz where P(true) = 0.5 for each question.

This means that

P(true) = 0.5

As a percentage,, we have

P(true) = 50%

When the probability of an event is 50%, then the event is equally likely and unlikely.

Hence, the liikelihood is (c) qually likely and unlikely.

Read more about probability at

https://brainly.com/question/24756209

#SPJ1

the cost of a luna bar is 2 dollars at harmons. what is the cost of 30 luna bars

Answers

Answer:

60

Step-by-step explanation:

2$ times 30 luna bars at harmons is $60

An ant crawled 2/8 yard from an ant mount. On which number line does point A represent the ants position after crawling 2/8 yard?

Answers

The number line on which point A represents the ant's position after crawling 2/8 yard would be a number line labeled in units of yards, with O representing 0 and A representing 1/4 yard to the right of O.

To decide the place of the insect subsequent to creeping 2/8 yard from the insect mount, we really want to address this distance on a number line.

Since 2/8 can be rearranged to 1/4, we can address the distance crept by the subterranean insect as 1/4 of a yard.

To develop a number line to address this distance, we can begin with the point addressing the subterranean insect mount (we should call it point O) and separate a distance of 1/4 of a yard to one side of O. This point, which we can name as point A, addresses the place of the insect subsequent to creeping 2/8 yard (or 1/4 yard) from the insect mount.

The number line would have units of yards, with the distance between every unit separated similarly. The point O would address 0 on the number line, and the point A would address 1/4 yard on the number line.

Thus, the number line on which point An addresses the insect's situation in the wake of creeping 2/8 yard would be a number line named in units of yards, with O addressing 0 and An addressing 1/4 yard to one side of O.

To learn more about number line, refer:

https://brainly.com/question/18586935

#SPJ1

The complete question is:

An ant crawled 2/8 yard from an ant mount. On which number line does point A represent the ants position after crawling 2/8 yard?

3. Find the solution to each equation mentally.
a. 30+ a = 40
b. 500+ b = 200
C. -1+c= -2
d. d +3.567=0

Answers

Answer:

[tex]a. \: 30 + a = 40 \\ a = 40 - 30 \\ a = 10 \\ b. \: 500 + b = 200 \\ b = 200 - 500 \\ b = - 300 \\ c. \: - 1 + c = - 2 \\ c = - 2 + 1 \\ c = - 1 \\ d. \: d +3.567=0 \\ d = 0 - 3.567 \\ d = - 3.567[/tex]

hope it helps:)

(3d). Mary decided to open a uniform cleaning service at GTUC. When she started the business she had to purchase an Ironing Board for $15 and an Iron for $35. Also, she figured it would cost $1.25 in cleaning products for each uniform, so she decided she was going to charge $3.25 for cleaning and pressing one entire uniform i. Write a system of equations that would represent the above scenario. ii. How many uniforms does Mary have to clean in order to break even? ​

Answers

she needs to clean 15 uniforms to break even

What expression is or is not equivalent, what is the answer?

Answers

Answer:
A and B

Step-by-step explanation:

-10/3= -3.33333333333

(- (-10))/(-3)= -10/3, (negative sign being erased)

=-3.33333333333

hope it helped:)

example of a multi-step combination problem

Answers

Answer:

A multi-step-word problem is like a puzzle with lots of pieces. Multi-step word problems are math problems that have more than one operation. An operation is addition, subtraction, multiplication, or division. Multi-step word problems can have any combination of these operations

Determine the point estimate of the population​ proportion, the margin of error for the following confidence​ interval, and the number of individuals in the sample with the specified​ characteristic, x, for the sample size provided.
Lower bound=0.640​, upper bound=0.910​, n=1200

Answers

The point estimate of the populationproportion, the margin of error for the following confidence​ interval, and the number of individuals in the sample are:

margin of error = 0. 135point estimate = 0. 775 x = 930 people

How to find the parameters ?

The point estimate of the population proportion:

= (lower bound + upper bound) / 2

= (0.640 + 0.910) / 2

= 0. 775

The margin of error for the confidence level :

= p - hat - lower bound

= 0. 775 - 0. 640

= 0. 135

The number of individuals in the sample with the specified characteristic (x ) :

= n x p - hat

= 1, 200 x 0. 775

= 930 people

Find out more on margin of error at https://brainly.com/question/30404882

#SPJ1

Please answer if you know this one with the steps thank you.

Answers

Step-by-step explanation:

T = 5200 = .2 ( E - 10600)

       5200 / .2 = E - 10600

            E = 5200/.2   + 10 600 = 36600 pounds

There are 6 bottles of water.
Each bottle is 1/2 full. If you
were to combine all the
water, how many full bottles
of water would there be?

Answers

Answer:3

Step-by-step explanation: 1/2 X 6 = 3

Answer:

6 x 1/2 = 3 full bottles

Step-by-step explanation:

Which of these expressions is equivalent to log (128^)?
OA. log (8) - log (12)
OB. 8 log (12)
C. log (8) log (12)
D. log (8) + log (12)
.

Answers

O think the answer is c

Use Chebyshev’s inequality and the value of W to decide whether there is statistical evidence, at the significance level of α=0.05 , that D, the average proportion of all lightbulbs that are defective, is greater than 0.10.

Answers

Chebyshev's inequality is a statistical tool that provides a bound on the probability that a random variable deviates from its mean by a certain amount. It can be used to determine whether there is statistical evidence to support a hypothesis based on sample data.

Suppose we have a random variable D that represents the proportion of defective lightbulbs in a population. We want to test the hypothesis that the average proportion of defective lightbulbs in the population is greater than 0.10, i.e., H₀: D ≤ 0.10 vs H₁: D > 0.10. Let's assume that we have a sample of n lightbulbs and that the sample proportion of defective lightbulbs is W.

Chebyshev's inequality states that for any random variable X, the probability that X deviates from its mean by k standard deviations is at most 1/k². In other words,

P(|X - μ| ≥ kσ) ≤ 1/k²,

where μ and σ are the mean and standard deviation of X, respectively.

Now, let's apply Chebyshev's inequality to our sample proportion W. Since we don't know the true mean and standard deviation of D, we can use the sample mean and sample standard deviation as estimates. The sample mean is W/n and the sample standard deviation is √[W(1-W)/n]. We want to find the probability that D is greater than 0.10, which is equivalent to finding the probability that W/n is greater than 0.10.

Let k = (0.10 - W/n)/(√[W(1-W)/n]). Then,

P(D > 0.10) = P(W/n > 0.10) = P(W - 0.10n > 0) = P(W - μ ≥ (0.10 - μ)n) ≤ σ²/[(0.10 - μ)n]²,

where μ = E(W) = D and σ² = Var(W) = D(1-D)/n. Thus,

P(D > 0.10) ≤ D(1-D)/n[(0.10 - D)n]².

To decide whether there is statistical evidence to support the hypothesis H₁, we need to compare this upper bound on the probability of D being greater than 0.10 to the significance level α = 0.05. If the upper bound is less than α, then we reject the null hypothesis H₀ in favor of the alternative hypothesis H₁.

To know more about Hypothesis here

https://brainly.com/question/29576929

#SPJ1

Other Questions
An artist buys 2 liters of paint for a project. When he is done with the project, he has 350 milliliters of the paint left over. The paint costs 2 per milliliter. How many dollars worth of paint does the artist use for the project? Louise stops at the gift store to buy a souvenir of the statue of liberty. the original height of the statue is 151 ft. if a scale factor of 1in = 20 ft is used to design the souvenir, what is the height of the replica? An electromagnetic wave traveling through a vacuumhas a wavelength of 1.5 101meter. What is theperiod of this electromagnetic wave? In Erikson's psychosocial theory of development, during a stage, a _____ must be successfully met to produce a _____ that can benefit the person from that point forward The following excerpt is dissonant. play play stop mute max volume 00:0002:49 audio selection select one: true false ) Which ratios have a unit rate of 3? Choose ALL that apply. 2 cups : 2 3 cup 314 3- cups: 2 cups 4 2 3 cup: 1 cup 1 1 cup: cup 15 K 1 2 cups cup 2 15 2 5 6 1 cups: 22 2 cups I need some assistance with this ? Please Pfizer pharmaceuticals is offering low-income senior citizens some of its most widely used prescriptions for $15 each a month--much below the regular costs for these drugs. This program to better serve senior citizens likely grew out of a __________ goal. Explain why the capitalist system developed in Europe. a portfolio's value increases by 17% during a financial boom and by 8% during normal times. it decreases by 12% during a recession. what is the expected return on this portfolio if each scenario is equally likely? explain how lung capacity related to the ability of the body to perform cellular respiration Jose Morales manages a large outdoor fruit stand in one of the less affluent neighbourhoods of San Jose, California. To replenish his supply, Jose buys boxes of the fruit early each morning from a grower south of San Jose. About 90 percent of the boxes of fruit turn out to be of satisfactory quality, but the other 10 percent are unsatisfactory. A satisfactory box contains 80 percent excellent fruit and will earn $200 profit for Jose. An unsatisfactory box contains 30 percent excellent fruit and will produce a loss of $1,000. Before Jose decides to accept a box, he is given the opportunity to sample one piece of fruit to test whether it is excellent. Based on that sample, he then has the option of rejecting the box without paying for it. Jose wonders (1) whether he should continue buying from this grower, (2) if so, whether it is worthwhile sampling just one piece of fruit from a box, and (3) if so, whether he should be accepting or rejecting the box based on the outcome of this sampling. Use the likelihoods/conditional and prior probabilities to calculate all the posteriors/revised probabilities. Show your work/calculations. (6 points) N. B. : Round the probabilities to two decimal points. Draw and solve the decision tree for this problem to determine the optimal decision strategy that Jose should follow. (Show your work/calculations). Verbally communicate the decision strategy. (25 points) ayuda porfa nose como se hace :'((((((((((((esta es la frmula: y=a(x-h)+k Akron, inc., owns all outstanding stock of toledo corporation. amortization expense of $15,000 per year for patented technology resulted from the original acquisition. for 2021, the companies had the following account balances: akron toledo sales $1,100,000 $600,000 cost of goods sold 500,000 400,000 operating expenses 400,000 220,000 investment income not given 0 dividends declared 80,000 30,000 intra-entity sales of $320,000 occurred during 2020 and again in 2021. this merchandise cost $240,000 each year. of the total transfers, $70,000 was still held on december 31, 2020, with $50,000 unsold on december 31, 2021. for consolidation purposes, does the direction of the transfers (upstream or downstream) affect the balances to be reported here? If two fair dice are rolled, what is the probability that the total showing is either even or less than five? 5.these are the choices:a. 5/9b. 17/36c. 19/36d. 11/18 Justin, Cam, and Ben are playing a board game where exactly one player will win. Ben estimates that Justin has a 20%20%20, percent chance of winning each game and that Cam has a 50%50%50, percent chance of winning each game. Urgenttttt what is true about the series given: 25+5+1+... the series converges to 31.25 the series diverges . the series converges to 125 the series does not converge or diverge . Your answer should be in the form p(x) +k/x+2 where p is a polynomial and k is an integer of x^2 +7x+12/x+2 mapa conceptual que relacione los tipos de dietas adaptndolas a la problemtica particular de cada persona. Which equation has a focus at (6, 12) and directrix of x = 12?1. ) ( y - 12)^2 = 1/12 ( x + 9 )2. ) ( y - 12 )^2 = -1/12 ( x + 9 )3. ) ( y - 12)^2 = 12 ( x+9 )4. ) ( y - 12)62 = -12 (x + 9 )Answer: C